Math, asked by Vanessa18, 1 year ago

There are 48 students in Mrs. Mitra's computer class. At present, the ratio of the number of students to the number of computers in her class is 4 : 1. How many MORE computers does she need to buy to make this ratio 3 : 1?
A. 1 B. 3
C. 4 D. 6

Answers

Answered by Vanshika666
24
Let students = 4x
computers = 1x

given total strength = 48
4x = 48
x=48/4
x=12
thus 1x = 12
No of computers is 12

New ratio = 3 : 1
Students = 3x
computers = 1x
3x = 48
x= 48 / 3
16
thus 1x = 16
more computers needed = 16-12 = 4
thus C. 4 is the correct answer
Answered by pulakmath007
6

4 computers Mrs. Mitra needs to buy to make this ratio 3 : 1

Given :

  • There are 48 students in Mrs. Mitra's computer class.

  • At present, the ratio of the number of students to the number of computers in her class is 4 : 1

To find :

The number of computers Mrs. Mitra needs to buy to make this ratio 3 : 1

A. 1

B. 3

C. 4

D. 6

Solution :

Step 1 of 2 :

Find number of computers at present

Here it is given that at present, the ratio of the number of students to the number of computers in her class is 4 : 1

Let number of students = 4x and number of computers = x

Now it is given that there are 48 students in Mrs. Mitra's computer class.

By the given condition

4x = 48

⇒ x = 12

The number of computers at present = 12

Step 2 of 2 :

Find the required number of computers

Let y computers Mrs. Mitra needs to buy to make this ratio 3 : 1

So by the given condition

48 : ( 12 + y ) = 3 : 1

\displaystyle \sf{ \implies  \frac{48}{12 + y}  =  \frac{3}{1} }

\displaystyle \sf{ \implies  36 + 3y = 48}

\displaystyle \sf{ \implies  3y = 48 - 36}

\displaystyle \sf{ \implies  3y = 12}

\displaystyle \sf{ \implies  y = 4}

4 computers Mrs. Mitra needs to buy to make this ratio 3 : 1

Hence the correct option is C. 4

━━━━━━━━━━━━━━━━

Learn more from Brainly :-

1. Ifa : b=c:d=e: f= 2:1, then (a + 2c + 3e) : (5b + 10d + 15f) is equal to:

https://brainly.in/question/23293448

2. If p/2q = 11/5 then p-2q/p+2q = ?

https://brainly.in/question/34239839

Similar questions